3
$\begingroup$

Wikipedia defined Resonance as the following : Resonance describes the phenomenon of increased amplitude that occurs when the frequency of an applied periodic force (or a Fourier component of it) is equal or close to a natural frequency of the system on which it acts. What is meant by : (or a Fourier component of it) ?

Wikipedia Link : https://en.wikipedia.org/wiki/Resonance

$\endgroup$
3
  • $\begingroup$ See: Fourier analysis $\endgroup$ Commented Jun 25, 2022 at 17:39
  • $\begingroup$ I did and I know what fourier analysis means but I'm not able to understand its major relationship with resonance actually. $\endgroup$ Commented Jun 25, 2022 at 17:57
  • 2
    $\begingroup$ The standard introduction to resonance considers $\ddot{x} + \omega_0^2 x = F \cos(\omega t)$ as $\omega \to \omega_0$. So now try changing the right hand side to $F_1 \cos(\omega_1 t) + F_2 \cos(\omega_2 t)$. If you still see resonance as one of those frequencies approaches $\omega_0$, that should answer your question. $\endgroup$ Commented Jun 25, 2022 at 18:12

1 Answer 1

1
+50
$\begingroup$

It means that the driving force is decomposed into sines and cosines and if one of this sinusoidal functions oscillates with frequency equal or close to the natural frequency, then the system resonates.

To see this, consider for example an oscillator described by $$\ddot x+2\gamma \dot x+\omega_0^2x=\frac{F(t)}{m},$$ when $F$ is an arbitrary periodic driving force, of period $T$. It can be written as a sum of sines and cosines through a Fourier decomposition, $$F(t)=\frac{a_0}{2}+\sum_{n=1}^\infty\left[ a_n\cos\left(\frac{2\pi nt}{T}\right)+b_n\sin\left(\frac{2\pi nt}{T}\right)\right].$$ Then the particular solution is $$x(t)=\frac{a_0}{2k}+\frac{F_0}{m}\sum_{n=1}^\infty\left[\frac{a_n\cos(2\pi nt/T)+b_n\sin(2\pi nt/T)}{\sqrt{(\omega_0^2-4\pi ^2n^2/T^2)^2+4\gamma^24\pi^2n^2/T^2}}\right],$$ and there is energy resonance when $2\pi n/T=\omega_0$ and amplitude resonance when $2\pi n/T=\sqrt{\omega_0^2-4\gamma^2}$. The amplitude of the $n$-mode of $x$ gets its maximum value or is close to it.

$\endgroup$
1
  • $\begingroup$ Exactly Answered My Question . I would like to gift you the bounty in 17 hours as the policy suggest with all thanks. $\endgroup$ Commented Jun 26, 2022 at 0:00

Your Answer

By clicking “Post Your Answer”, you agree to our terms of service and acknowledge you have read our privacy policy.

Not the answer you're looking for? Browse other questions tagged or ask your own question.